Find the slope
of the line passing through the points (3, 4)
and
(8, -3).

Answers

Answer 1

Answer:

-7/5

Step-by-step explanation:

I think let me know

Answer 2

Answer:

7/-5  or -7/5

Step-by-step explanation:

This shall be quite an easy problem, I shall be doubting that this is high school, however, I am happy to aid :)

We shall begin by labeling the points given to us to prepare for inputting the values in the slope formula

(3,4).     (8,-3)

x1,y1     x2,y2

Slope Formula:

y1 - y2

x1 - x2

Inputting the values:

4 - (-3)

3 - 8

Solve:

7

-5

The slope of the line passing through the points (3,4) and (8,-3) shall be 7/-5 or -7/5 negatives shall go both ways of fractions


Related Questions

What is the difference between-5and2

Answers

Answer:

7

Step-by-step explanation:

Consider the absolute value of the difference , that is

| - 5 - 2 | = | - 7 | = 7

or

| 2 - (- 5) | = | 2 + 5 | = | 7 | = 7

Answer:

7

Step-by-step explanation:

Difference is - sign so the equation is: 2- -5 which is 7. Or

think a number line, -5 is 5 spots to 0, then two more spots to 2 so 5+2=7

Find m/ELM if m/ELM = 15x - 1, m/KLE = 20°, and m/KLM = 17x - 1.​

Answers

Answer:

∠ ELM = 149°

Step-by-step explanation:

∠ KLM = ∠ KLE + ∠ ELM  , substitute values

17x - 1 = 20 + 15x - 1

17x - 1 = 15x + 19 ( subtract 15x from both sides )

2x - 1 = 19 ( add 1 to both sides  )

2x = 20 ( divide both sides by 2 )

x = 10

Then

∠ ELM = 15x - 1 = 15(10) - 1 = 150 - 1 = 149°


What is the surface area of the right prism?
92 ft2
46 ft2
48 ft2
70 ft2

(will mark brainliest <3)

Answers

Answer:  70 ft^2   ( choice D )

=========================================================

Work Shown:

L = 8 ft = lengthW = 3 ft = widthH = 1 ft = height

SA = surface area of the rectangular prism (aka block or box)

SA = 2*(LW + LH + WH)

SA = 2*(8*3 + 8*1 + 3*1)

SA = 2*(24 + 8 + 3)

SA = 2*(35)

SA = 70 square feet

This is the amount of wrapping paper you would need to cover all six sides of the box. This assumes that there are no gaps or overlaps.

which statement is true

Answers

3) an $8 delivery fee and $1.50 per litre of water

This is because the $8 is a constant baseline, then adding $1.50 times the amount of litres purchased.

A game involves correctly choosing the 5 correct numbers from 1 through 18 that are randomly drawn. What is the probability that a person wins the game, if they enter a) once? b) 7 times with a different choice each time?

Answers

Answer:

[tex]=\frac{1}{8568}\ = .00011\\\ =\frac{7}{8568} = .00081[/tex]

Step-by-step explanation:

[tex]5/18\cdot \:4/17\cdot \:3/16\cdot \:2/15\cdot \:1/14=\frac{1}{8568}[/tex]

Match the answers……………..

Answers

9 in 8956 = 900

9 in 95675 = 90000

9 = 9 in 124569

9 in 68795 = 90

90000 = 9 in 2549652.........

hope it helps...

working alone, aliyah can dig a 10ft by 10 ft hole is 6 hours. one day her friend eugene helped her and it only took 3.75 hours. how long would it take eugene to do it alone? disclaimer: the answer is 11.95 hours, but i keep getting 10. please use the 1/x+1/y=1/z strategy

Answers

Answer:

I think the answer is 10 hours

the 11.95 seems wrong

Step-by-step explanation:

[tex]\frac{1}{6} + \frac{1}{y} = \frac{1}{3.75}[/tex]

multiply by 22.5 y

3.75 y + 22.5 = 6y

y = 10

Write down at least five number pairs to solve the equation vw = 50

Answers

Answer:

1, 50

2, 25

5, 10

10, 5

25, 2

Answer:

Step-by-step explanation:

V  W  

1 x 50     = 50

1.5 x 33 1/3 = 50

2 x 25     = 50

2.5 x 20     = 50

3 x 16 2/3 = 50

3.5 x 14 2/7 = 50

4 x 12 1/2 = 50

4.5 x 11 1/9 = 50

5 x 10     = 50

5.5 x 9     = 50

6 x 8 1/3 = 50

6.5 x 7 2/3 = 50

7 x 7 1/7 = 50

7.5 x 6 2/3 = 50

8 x 6 1/4 = 50

8.5 x 5 8/9 = 50

9 x 5 5/9 = 50

9.5 x 5 1/4 = 50

10 x 5     = 50

10.5 x 4 3/4 = 50

11 x 4 5/9 = 50

11.5 x 4 1/3 = 50

12 x 4 1/6 = 50

12.5 x 4     = 50

13 x 3 6/7 = 50

13.5 x 3 5/7 = 50

14 x 3 4/7 = 50

if f(x) = 2x²+2x-1, g(x) = x²+5x+2 f(x)=g(x), find the value of x​

Answers

Answer:

Possible Answers: –5. –21. 7. –1

Answer:

just put the values and simpilyfy it untill you get the value of X

hence, the value of X is either 3 or, -1

91. Jack can read 45 pages of his book in one and a
half hours. At that rate, how long will it take him to
read the entire 300-page book?

Answers

Answer:

10 hours

Step-by-step explanation:

45 pages per 1 hour and 30 minutes,  or 60 min +30 min =90 min

90 minutes / 45 pages has to be equal to an equivalent fraction where we have 300 pages

90 minutes /45 pages = ? minutes / 300 pages , multiply both sides by 300

? minutes = 90*300/45 = 600 minutes

to read 300-page book will take 600 minutes = 10 hours

Someone please help me find y if you don’t mind Thankyouu so much

Answers

Answer:

Step-by-step explanation:

Decreasing rate = 12% = 0.12

y = 2400 * (1- 0.12)^x = 2400*(0.88)^x

x = 8 years

[tex]y = 2400 *(0.88)^{8}\\\\= 2400*0.36\\\\= 863[/tex]

Find the cross product (7,9, 6)x 44, 1, 5). Is the resulting vector perpendicular to the given vectors?
a. (-57, 43, 33); yes
c. (33. – 59, - 57); no
b. (39, – 59,43), yes
d. (33, - 57, 49); no

Answers

Answer:

Option B, (39, -59, 43)

Answered by GAUTHMATH

Find the place value of 8 in 2456.1387.
Tenths
Hundredths
Thousandths
Thousands

Answers

Answer: Third Choice. Thousandths

Step-by-step explanation:

Concept:

Here, we need to know the order and name of each place value.

Please refer to the attachment below for the specified names.

Solve:

STEP ONE: Orde and name each place

2 ⇒ One Thousands

4 ⇒ Hundreds

5 ⇒ Tens

6 ⇒ Ones

.

1 ⇒ Tenths

3 ⇒ Hundredths

8 ⇒ One Thousandths

7 ⇒ Ten Thousandths

STEP TWO: Find the number [8] in the number

As we can see from the list above, 8 is at the right of the decimal point, thus, the place value is Thousandths.

Hope this helps!! :)

Please let me know if you have any questions

find the radius of a circle for which an arc 6 cm long subtends an angle of 1/3 radians at the center?
plz some one can help to solve the question??​

Answers

Step-by-step explanation:

Eueydhhdgdgdbdbddbdbhd

Answer:

Hello,

[tex]R=\dfrac{18}{\pi}\ (cm)[/tex]

Step-by-step explanation:

[tex]Formula: \ L=\theta*R\\[/tex]

[tex]R=\dfrac{6}{\dfrac{\pi}{3} } =\dfrac{6*3}{\pi} =\dfrac{18}{\pi}\ (cm)[/tex]

What is
f(x)=(x-2)(x-6) in standard form

Answers

Standard form is
y
=
x
2

4
x

12
Explanation:
This is a function of degree
2
i.e. quadratic equation and its standard firm is
y
=
a
x
2
+
b
x
+
c
. Hence, the standard form for
y
=
(
x
+
2
)
(
x

6
)
is
=
x
(
x

6
)
+
2
(
x

6
)
=
x
2

6
x
+
2
x

12
=
x
2

4
x

12

but this is for x+2 instead of x-2

What is the perimeter of the right triangle with legs (2x + 1) feet and (4x - 4) feet and hypotenuse (4x - 1) feet? Give your answer in terms of x in the simplest form.

Answers

Answer:

10x-4 feet

Step-by-step explanation:

The perimeter is the amount of the sides together so add the three sides together

2x+1+4x-4+4x-1

Combine like terms

10x-4

(You can also factor out 2 but that would not be simplest --> 2(5x-2))

9)Using appropriate properties , find 7
5
×
5
12

3
12
×
7
5

1
15

Answers

Answer:

-20940

Step-by-step explanation:

5 x 512 - 312 x 75 - 115

According to BODMAS ,

2575 - 23400 - 115

= -2090

Answer:

[tex] \frac{1}{6} [/tex]

Step-by-step explanation:

[tex] \frac{7}{5} \times \frac{5}{12} - \frac{3}{12} \times \frac{7}{5} - \frac{1}{15} = \frac{7}{5} ( \frac{5}{12} - \frac{3}{12} ) - \frac{1}{15} = \frac{7}{5} \times \frac{1}{6} - \frac{1}{15} = \frac{7}{30} - \frac{2}{30} = \frac{5}{30} = \frac{1}{6} [/tex]

HELP PLEASE
i would really appreciate if someone answered this correctly!

Answers

Answer:

n-32

Step-by-step explanation:

edmentum

2.
Lucy shares 42 marbles between her and her 6 friends. How many does each child?

Answers

42 marbles divided by 7 (because lucy has 6 friends but she has to add herself because they all are sharing the 42 marbles between each other) = 6 marbles goes to each child


6 marbles

HELP ME PLSSSSS I NEED HELP PLS PLS PLS ITS PYTHAGOREAN THEOREM

Answers

H=9ftB=6ft

Using Pythagorean thereon

[tex]\\ \Large\sf\longmapsto P^2=H^2-B^2[/tex]

[tex]\\ \Large\sf\longmapsto P^2=9^2-6^2[/tex]

[tex]\\ \Large\sf\longmapsto P^2=81-36[/tex]

[tex]\\ \Large\sf\longmapsto P^2=45[/tex]

[tex]\\ \Large\sf\longmapsto P=\sqrt{45}[/tex]

[tex]\\ \Large\sf\longmapsto P=6.3ft[/tex]

HELP ASAP LIKE PLEASE

Answers

Answer: (3b+2)(b+2)

Step-by-step explanation:

The data set represents the total number of pencils each student in a class needs to sharpen.

0, 1, 1, 1, 2, 3, 4, 4, 6, 6, 9

Which box plot correctly represents the data?

A number line goes from 0 to 10. The whiskers range from 0 to 9, and the box ranges from 2 to 6. A line divides the box at 3.
A number line goes from 0 to 10. The whiskers range from 0 to 9, and the box ranges from 2 to 6. A line divides the box at 4.
A number line goes from 0 to 10. The whiskers range from 0 to 9, and the box ranges from 1 to 6. A line divides the box at 4.
A number line goes from 0 to 10. The whiskers range from 0 to 9, and the box ranges from 1 to 6. A line divides the box at 3.

Answers

Given:

The data set is:

0, 1, 1, 1, 2, 3, 4, 4, 6, 6, 9

To find:

The correct statement that represents the box plot of the data correctly.

Solution:

We have,

0, 1, 1, 1, 2, 3, 4, 4, 6, 6, 9

Divide the data in 2 equal parts.

(0, 1, 1, 1, 2), 3, (4, 4, 6, 6, 9)

Divide each parenthesis in 2 equal parts.

(0, 1), 1, (1, 2), 3, (4, 4), 6, (6, 9)

Here,

Minimum value = 0

First quartile = 1

Median = 3

Third quartile = 6

Maximum value = 9

A number line goes from 0 to 10. The whiskers range from 0 to 9, and the box ranges from 1 to 6. A line divides the box at 3.

Therefore, the correct option is D.

Answer:

its d

Step-by-step explanation: my yt Tray Clayps

Find C to two decimal places and find the measurement of angle be use your calculator

Answers

Let's solve

P=8B=5

Using Pythagorean theorem

[tex]\\ \sf\longmapsto H^2=P^2+B^2[/tex]

[tex]\\ \sf\longmapsto c^2=8^2+5^2[/tex]

[tex]\\ \sf\longmapsto c^2=64+25[/tex]

[tex]\\ \sf\longmapsto c^2=89[/tex]

[tex]\\ \sf\longmapsto c=\sqrt{89}[/tex]

[tex]\\ \sf\longmapsto c\approx 9.22[/tex]

which choice is equivalent to the expression below? 7x√2 - 4√2 + x√2

A. 6x√2 - 3√2
B. 5x^2√2
C. 3x√2
D. 8x√2 - 4√2

Answers

Answer:

D

Step-by-step explanation:

[tex]8x \sqrt{2} - 4 \sqrt{2} [/tex]

Convert 5π∕6 radians to degrees. Question 1 options: A) 25° B) 150° C) 150π° D) 1080°

Answers

Step-by-step explanation:

Hi there!

Given;

= 5(π\6)

We have;

π = 180°

Keeping value of π in the question;

= 5(180°/6)

= 5*30°

= 150°

Therefore, answer is option B.

Hope it helps!

Rob cuts a circular hole out of a rectangular piece of paper. The paper measures 20 centimeters by 30 centimeters. The hole is 10 centimeters in diameter. How much of the piece of paper, in square centimeters, is left over after the hole is cut out?

Answers

Answer:

521.25

Step-by-step explanation:

the circle is 78.75 in area and the square is 600 so 600- 78.75 = 521.25

please answer this question!!​

Answers

Answer:

a

Step-by-step explanation:

all angles of an equilaterall triangle are equal therefore 180÷3 = 60

If a wheel has a radius of 5cm
how much is one rotation of the wheel
How many rotations can the wheel do within a distance of 50km

Answers

Answer:

circumference = 2*PI*radius

circumference = 2 * PI * 5 cm

circumference = 31.4159265358979 cm

50 km = 500,000 centimeters

rotations = 500,000  / 31.4159 cm

15,915.51  rotations

Step-by-step explanation:

equivalent expression: 3 + 4(2z - 1)

Answers

Answer:

8z - 1

Step-by-step explanation:

Given

3 + 4(2z - 1) ← multiply each term in the parenthesis by 4

= 3 + 8z - 4 ← collect like terms

= 8z - 1

Answer:

-1 + 8z

Step-by-step explanation:

First use the distributive property of multiplication (Just multiply 4 with all numbers in the parenthesis):

3 + 4(2z - 1)

3 + 8z - 4

Group like terms:

3 + 8z - 4

-1 + 8z

The answer is -1 + 8z

Hope this helped.

Jim removed 27 gallons of water from a rainwater storage tank. There are 59 gallons left in the tank. What equation can Jim use to find how much water was in the tank earlier? Use x to represent the amount of water originally in the tank.

Answers

Answer: X = 86 gallons

Step-by-step explanation:

X-27=59

X=59+27

Other Questions
I need help with this please!!!! How do you know that a liquid exerts pressure? Find two consecutive even numbers whose sum is 758. Write a program that asks the user to enter in a username and then examines that username to make sure it complies with the rules above. Here's a sample running of the program - note that you want to keep prompting the user until they supply you with a valid username: help again geometry similarity transformations describe how Ubuntu could help fight poverty How long would it take the car to travel a distance of 2 meters at this average speed? Which item is an example of a secondary source? derive an expression for resistivity of conductor of length l and area of cross section A Simplify the given expression. Fill in the blank in the following sentence with the appropriate form of theverb faire below.Ils____du ski?A. faitB. faisons C. faitesD. font Which was a common restriction included in the black codes? 4. . 2 . . how many square metres of floor are there in a room of 6 metres ig something like that Use what you know about sine, cosine, and tangent to calculate the height of the buildings in the diagram below. An advertiser goes to a printer and is charged $36 for 80 copies of one flyer and $46 for 242 copies of another flyer. The printer charges a fixed setup cost plus a charge for every copy of single-page flyers. Find a function that describes the cost of a printing job, if xx is the number of copies made. PLEaseeee HELP MEEE I beg pleaseee I think I got this wrong please correct me. He said to me, "Do you like ice-cream?" (Change the sentence into reported speech In figure above, if l1 | | l2 then value of x is:a) 40b) 50c) 80d) 100 solve for x. solve for x. solve for x.